Cono 2022 P3

Problemas que aparecen en el Archivo de Enunciados.
Avatar de Usuario
Gianni De Rico

FOFO 7 años - Mención Especial-FOFO 7 años OFO - Medalla de Oro-OFO 2019 FOFO 9 años - Jurado-FOFO 9 años COFFEE - Jurado-COFFEE Matías Saucedo OFO - Jurado-OFO 2020
FOFO Pascua 2020 - Jurado-FOFO Pascua 2020 COFFEE - Jurado-COFFEE Carolina González COFFEE - Jurado-COFFEE Ariel Zylber COFFEE - Jurado-COFFEE Iván Sadofschi FOFO 10 años - Jurado-FOFO 10 años
OFO - Jurado-OFO 2021 FOFO 11 años - Jurado-FOFO 11 años OFO - Jurado-OFO 2022 FOFO Pascua 2022 - Jurado-FOFO Pascua 2022 FOFO 12 años - Jurado-FOFO 12 años
OFO - Jurado-OFO 2023 FOFO 13 años - Jurado-FOFO 13 años OFO - Jurado-OFO 2024 FOFO Pascua 2024 - Jurado-FOFO Pascua 2024
Mensajes: 2222
Registrado: Vie 16 Sep, 2016 6:58 pm
Medallas: 19
Nivel: Exolímpico
Ubicación: Rosario
Contactar:

Cono 2022 P3

Mensaje sin leer por Gianni De Rico »

Demuestre que para todo entero positivo $n$, existe un entero positivo $k$, de modo que cada uno de los números $k,k^2,\ldots ,k^n$ tenga al menos un bloque $2022$ en su representación decimal.
Por ejemplo, los números $4\textbf{2022}13$ y $544\textbf{2022}1\textbf{2022}$ tienen al menos un bloque $2022$ en su representación decimal.
♪♫ do re mi función lineal ♪♫
Fedex

COFFEE - Mención-COFFEE Matías Saucedo OFO - Medalla de Plata-OFO 2020 FOFO Pascua 2020 - Medalla-FOFO Pascua 2020 COFFEE - Mención-COFFEE Ariel Zylber COFFEE - Mención-COFFEE Iván Sadofschi
FOFO 10 años - Medalla-FOFO 10 años OFO - Medalla de Plata-OFO 2021 OFO - Jurado-OFO 2022 OFO - Jurado-OFO 2023 FOFO 13 años - Jurado-FOFO 13 años
OFO - Jurado-OFO 2024
Mensajes: 272
Registrado: Mar 31 Dic, 2019 2:26 am
Medallas: 11
Nivel: 3
Ubicación: Rosario, Santa Fe
Contactar:

Re: Cono 2022 P3

Mensaje sin leer por Fedex »

Uno de los mejores problemas :mrgreen:
Spoiler: mostrar
Vamos a demostrarlo por inducción en $n$:
Para el caso base, $n=1$, podemos tomar $k=2022$.
Ahora viene lo interesante, sea $k$ un número que verifica el enunciado para $n$. Para $n+1$ tomamos un número de la forma $k’ = k + 10^x \cdot c$ donde elegiremos $x$ y $c$ (enteros positivos) a placer de forma que se den las siguientes cosas:

En primer lugar, $x$ es lo suficientemente grande para que $(k + 10^x \cdot c)^i$ termine en $k^i$ para todo $1 \leq i \leq n$, en este caso bastará tomar un $x \geq d(k^n)$ donde $d(y)$ es la cantidad de dígitos de $y$, de esta forma:
$$(k + 10^x \cdot c)^i \equiv k^i \; (10^{d(k^n)})$$
Y como $k^i < 10^{d(k^n)}$ nos aseguramos que estas potencias $i$-esimas tienen un bloque $2022$ al terminar en $k^i$.

En segundo lugar, sea $m(j) = \binom{n+1}{j} \cdot c^j \cdot k^{n+1-j}$ luego:
$$(k + 10^x \cdot c)^{n+1} = \sum_{j=0}^{n+1} 10^{xj} \cdot m(j)$$
Ahora, como entre $10^{xj}$ y $10^{x(j-1)}$ hay $x$ dígitos de diferencia también podemos exigirle a este $x$ ser lo suficientemente grande para que los $m(j)$ no se me “superpongan” al desarrollar a este número en base $10$. Bastará tomar $x \geq d(max(m(j)))$.

En tercer lugar, quiero que $c^{n+1}$ empiece con un bloque de $2022$ en cuyo caso la potencia $n+1$-esima que tenemos empezará de esta manera, cerrando el paso inductivo. Para ello me gustaría que $c$ verifique:
$$2022 \cdot 10^h \leq c^{n+1} \leq 2023 \cdot 10^h \Rightarrow 2022^{\frac{1}{n+1}} \cdot 10^{\frac{h}{n+1}} \leq c \leq 2023^{\frac{1}{n+1}} \cdot 10^{\frac{h}{n+1}} $$
Con lo que me bastará tomar un $h$ lo suficientemente grande para que:
$$10^{\frac{h}{n+1}} \cdot (2023^{\frac{1}{n+1}} - 2022^{\frac{1}{n+1}}) > 1$$
Apareciendo en este intervalo un $c$ entero que verifica esa propiedad.

Y estamos, vimos que podemos tomar $x$ y $c$ a conveniencia para que exista este $k’$ para $n+1$, con lo que queda demostrado el problema para todo $n$ entero positivo.
This homie really did 1 at P6 and dipped.
Mijail
Mensajes: 21
Registrado: Mié 11 Oct, 2017 9:56 pm
Nivel: 3
Ubicación: Peru, Lima

Re: Cono 2022 P3

Mensaje sin leer por Mijail »

Una solucion algo diferente... :)
Spoiler: mostrar
Clave del problema induccion en $n$.
Si $k$ cumple para $n$ entonces cambie a $A.10^r+k = k'$ usando binomio de newton tomando $r$ muy grande tenemos que el bloque de digitos de $k^i$ y $A.10^r.k^{i-1}.(i)$ siempre aparezca en la expresion decimal de $(k')^i$ para $i=1,2,\dots,n+1$. Entonces usando la definicion de $k$ basta analizar solo el caso $i=n+1$.

Entonces usamos el $A.10^r.k^{n}.(n+1)$ queremos que tenga un bloque de $2022$ pero esto es un hecho conocido ya que todo numero tiene un multiplo asi, solo elija el $A$ adecuado para generar ese multiplo.
Spoiler: mostrar
Identico a lo que se hace para que un numero tenga un multiplo formado solo por unos y ceros. :mrgreen: (Palomar)
Responder